User avatar
 
smiller
Thanks Received: 73
Atticus Finch
Atticus Finch
 
Posts: 205
Joined: February 01st, 2013
 
This post thanked 1 time.
 
 

Q10 - If Thompson appeals to moderates

by smiller Mon Nov 06, 2017 3:45 am

Question Type:
Match the Reasoning

Stimulus Breakdown:
Premise: Thompson appeals to moderates → ardent supporters desert AND he won't win the election
Premise: Thompson doesn't appeal to moderates → moderates vote for opponent AND he won't win the election
Conclusion: Either way, Thompson won't win the election

Answer Anticipation:
We're looking for an answer that matches the structure of the stimulus. Here's the structure in more generic terms:

A → B and C
~ A → D and C
Therefore, A or ~ A → C

It's interesting to note that the end result in each premise is "C," (Thompson won't win the election), and the conclusion focuses on that end result. It actually looks like each premises is presenting a cause and effect:

A → B, causing C
~ A → D, causing C

We should keep this in mind as we look at answers.

Correct answer:
(B)

Answer choice analysis:
(A) Premise Mismatch: The second premise in this answer is ~ A → ~ B. To be even close, it would need to be ~ A → B. The conclusion also does not match.

(B) Correct:
A → B, causing C
~ A → D, causing C
Therefore, A or ~ A → C

(C) Premise Mismatch: The premise structure in this argument is:
~A → B
B → C
The conclusion also does not match.

(D) Premise Mismatch: The premise structure in this argument is:
A → B
C → B and D

(E) Conclusion Mismatch: The premises in the argument are about having an effective spokesperson, but the conclusion shifts to a claim about the success of the project. We don't know that having an effective spokesperson will, by itself, make the project successful.

Takeaway/Pattern:
As is usually the case with Matching questions, we can look for answer choices with definite mismatches and eliminate those to find the right answer. In this question, whether you interpret each premise in the stimulus as having two separate outcomes, or as showing a cause and effect, answer choice (B) is the closest match. All of the other answers contain significant mismatches.

#officialexplanation